¿La relación de conmutación canónica fija la forma del operador de cantidad de movimiento?

Para la mecánica cuántica unidimensional

[ X ^ , pag ^ ] = i .

¿Fija esto unívocamente la forma de la pag ^ ¿operador? Mi apuesta es no porque pag ^ en realidad depende de si estamos en una representación coordinada o de momento, pero no sé si esa afirmación constituye una prueba. Además, si elegimos X ^ ψ = X ψ ¿La respuesta de la siguiente pregunta es afirmativa?

para el segundo

( X ^ pag ^ pag ^ X ^ ) ψ = X pag ^ ψ pag ^ X ψ = i ψ ,

pero no veo como puedo decir eso pag ^ debe ser proporcional a X . no se si tratando de ver eso pag ^ debe satisfacer la regla de Leibniz y por lo tanto debe ser proporcional a la X derivado podría ayudar. O usando el hecho de que X ^ y pag ^ debe ser hermitiano

Cualquier pista será apreciada.

Véase, por ejemplo, el teorema de Stone-von Neumann en Wikipedia .
No puede, porque, entre otras cosas, tenemos [ X ^ , y ^ ] = [ y ^ , pag ^ X ] = 0 , entonces si definimos PAG ^ pag ^ + y ^ , entonces también tenemos [ X ^ , PAG ^ ] = [ X ^ , pag ^ X ] = i

Respuestas (3)

Ya tiene respuestas "prácticas", por lo que tengo la intención de responder desde otro punto de vista.

Hay un teorema bastante famoso debido a Stone y von Neumann, luego mejorado por Mackay, y finalmente por Dixmier y Nelson, que a grandes rasgos establece el siguiente resultado dentro de la versión más elemental. (Otra versión del teorema se centra en los grupos unitarios generados por X y PAG evitando problemas con los dominios, sin embargo, me quedo aquí con la versión del operador autoadjunto).

TEOREMA. (declaración aproximada "para físicos") Si tiene un par de operadores autoadjuntos X y PAG definido en un espacio de Hilbert H tales que se conjugan entre sí:

[ X , PAG ] = i yo ( 1 )
y hay un vector cíclico para X y PAG , entonces existe un operador unitario tu : L 2 ( R , d X ) H tal que:

( tu 1 X tu ) ψ ( X ) = X ψ ( X ) y ( tu 1 PAG tu ) ψ ( X ) = i d ψ ( X ) d X . ( 2 )

(La declaración rigurosa, en esta versión similar a Nelson, se lee como sigue

TEOREMA. Dejar X y PAG ser un par de operadores autoadjuntos en un espacio complejo de Hilbert H tal que (a) verifican (1) en un subespacio denso invariante común S H , (b) X 2 + PAG 2 es esencialmente autoadjunto en S y (c) todos los vectores en S son cíclicos para X y PAG . Entonces existe un operador unitario tu : L 2 ( R , d X ) H tales que (2) son válidos para ψ C 0 ( R ) .

Tenga en cuenta que los operadores definidos en los lados derechos de (2) admiten extensiones autoadjuntas únicas, por lo que fijan completamente los operadores que representan los observables respectivos. Podemos reemplazar igualmente C 0 ( R ) para el espacio de Schwartz S ( R ) en la última declaración).

Salvo tecnicismos, todo eso significa que las relaciones de conmutación en realidad fijan los observables de posición y momento, así como el espacio de Hilbert. Por ejemplo, refiriéndose a la respuesta de Murod Abdukhakimov, si la adición de F a las expresiones estándar de X y PAG da lugar a operadores verdaderamente autoadjuntos, luego una transformación unitaria (solo esa conexión ψ a ψ en la respuesta de Murod Abdukhakimov) se deshace de la deformación restaurando la expresión estándar. Recuerde que las transformaciones unitarias no alteran todos los objetos físicos de QM.

El resultado se extiende a R norte , es decir, en relación con las partículas en el espacio para norte = 3 . Eliminando el requisito de irreductibilidad, la tesis se mantiene de todos modos, pero H descomponga en una suma directa (¡no integral directa!) de subespacios cerrados donde la declaración fuerte es válida.

Hay importantes consecuencias de este teorema fundamental. Ante todo H debe ser separable como L 2 ( R , d X ) es. Además no hay operador de tiempo T (conjugado con el operador hamiltoniano H ) existe si el identificador del operador hamiltoniano está acotado a continuación como requiere la física. Esta última afirmación se debe al hecho de que el teorema fija los espectros de X y PAG como los ejes reales enteros en ambos casos, de modo que el espectro de H no estaría acotado por debajo si T , H eran un par conjugado de operadores. Surge un teorema de no-go similar con respecto a la cuantización de una partícula en un círculo cuando se intenta definir operadores autoadjuntos de posición e impulso. El intento de resolver estos resultados fallidos dio lugar a una formulación más general de la mecánica cuántica basada en la noción de POVM y finalmente resultó ser muy útil en otros contextos como la teoría cuántica de la información.

Una observación importante es que el resultado de Stone-von Neumann - MacKay - Dixmier -Nelson falla cuando se trata de sistemas de dimensiones infinitas. Es decir, en términos generales, pasando del (espacio simpléctico) de un número finito de partículas al (espacio simpléctico) de un campo. En ese caso, las relaciones canónicas de conmutación de X i y PAG j son reemplazados por los de los campos cuánticos. P.ej:,

[ ϕ ( t , X ) , π ( t , y ) ] = i d ( X , y ) yo

o versiones más sofisticadas de ellos. En esta coyuntura existen infinitas representaciones del álgebra de observables que no pueden ser conectadas por operadores unitarios. Este es un fenómeno bien conocido en QFT o mecánica estadística cuántica (en el límite termodinámico). Por ejemplo, la teoría libre y la teoría de la interacción de un campo cuántico dado no pueden representarse en el mismo espacio de Hilbert una vez que se asumen requisitos estándar sobre estados y observables (el llamado teorema de Haag y esta es la razón profunda por la que el formalismo LSZ utiliza la topología débil en lugar del fuerte como en la teoría cuántica estándar de la dispersión).

Si se incluyen cargas de superselecciones en el álgebra de observables, automáticamente surgen representaciones no unitariamente equivalentes del álgebra dando lugar a sectores.

En QFT en espaciotiempo curvo la aparición de representaciones no equivalentes del álgebra de observables es un fenómeno bastante común debido a la presencia de curvatura del espaciotiempo.

+1. Tengo curiosidad por saber exactamente cómo LSZ usa una topología débil en lugar de una fuerte, ya sabes, a los libros de física generalmente no les importa. ¿Alguna buena referencia?
Tal vez puedas encontrar este comentario en el libro de texto de Haag. Creo que encontré este comentario hace muchos años en un libro de Hepp. Sin embargo, el problema es el llamado teorema de Haag, los operadores formales de Moller no pueden definir una transformación unitaria del espacio de Hilbert de la teoría libre al de la interactuante sin violar el teorema de Haag. Entonces, el límite debe calcularse para los elementos de la matriz, como de hecho se hace en el formalismo LSZ.
No estoy muy familiarizado con las condiciones (b) y (c) del riguroso enunciado del teorema anterior. ¿Puede hablar en absoluto de por qué estas son condiciones naturales para imponer a X y PAG ? (También estoy oxidado con la definición de "cíclico").

No. Puede agregar un cambio constante arbitrario (o un operador arbitrario que se desplaza con X ) sin afectar el CCR.

Para QM unidimensional, la solución general del CCR con X ^ representado como multiplicación por X en funciones de onda con argumento X es pag ^ = pag ^ 0 A ( X ^ )     , donde pag ^ 0 es el operador de momento canónico , y A ( X ) es una función arbitraria de X .
Prueba. La diferencia A ^ := pag ^ 0 pag ^   viaja con X ^ , por lo tanto es una función de X ^ .

En un caso más general podría ser:

pag X = i h X + F X

pag y = i h y + F y

pag z = i h z + F z

donde F ( X , y , z ) - función arbitraria.

Pero también puedes reemplazar la función de onda ψ = mi i h F ( X , y , z ) ψ lo que te lleva de vuelta a la forma tradicional.

Parece una transformación de calibre, ¿no?

Más directamente, corresponde a una transformación canónica con F correspondiente a la función generadora de tipo 1 Yo diria.